Probability Density of ##x## (Wiener Process)

Click For Summary
SUMMARY

The discussion focuses on determining the probability density function P(x) for a Wiener process W(t). The user derives P(x) using variable transformation and integration, concluding with the expression $$P(x)=\frac{e^{lnx/{2bt}}2\sqrt{blnx}}{x\sqrt{2{\pi}t}}$$. However, it is clarified that P(x) is not Gaussian due to the non-linear transformation of the Gaussian variable W_t. A recommended approach is to calculate the cumulative probability function first and then differentiate it to obtain the probability density function.

PREREQUISITES
  • Understanding of Wiener processes and stochastic calculus
  • Familiarity with probability density functions and cumulative distribution functions
  • Knowledge of variable transformation techniques in probability
  • Basic concepts of Ito's Lemma (optional for advanced understanding)
NEXT STEPS
  • Study the derivation of cumulative probability functions for non-linear transformations
  • Learn about the properties and applications of Ito's Lemma in stochastic processes
  • Explore the implications of Gaussian versus non-Gaussian distributions in probability theory
  • Investigate numerical methods for approximating probability densities in complex systems
USEFUL FOR

Mathematicians, statisticians, and researchers in fields involving stochastic processes, particularly those working with Wiener processes and probability theory.

lelouch_v1
Messages
9
Reaction score
0
Homework Statement
Given ##x(t)=\exp{-b(W(t))^2}##, where ##W(t)## is a Wiener process, solve the below questions:
i)What values can x take?
ii)What is the probability density for x?
Relevant Equations
##P(W)=\frac{\exp{-W^2/(2t)}}{\sqrt{2\pi t}}##
Suppose that W(t) is just a Wiener process (i.e. a Gaussian in general). I want to know what the probability density for x, P(x), is. I started off by just assuming that I want to measure the expectation value of an observable f(x), so ##<f(x)>=\int_{W=0}^{W=t}{P(W)f(g(W))dW} \ \ ,\ \ x=g(W) ## Then I transformed variables from W to t and I got $$<f(x)>=\int_{x=g(0)}^{x=g(t)}P(g^{-1}(x))f(x)(\frac{dW}{dx})dx=\int_{x=g(0)}^{x=g(t)}\frac{P(g^{-1}(x))}{g'(g^{-1}(x))}f(x)dx$$ so I just assume that the probability for x is $$P(x)=\frac{P(g^{-1}(x))}{g'(g^{-1}(x))}$$ Since x=g(W), then $$W=g^{-1}(x)$$, but from (1) I get $$\frac{dW}{dx}=\frac{1}{2x\sqrt{blnx}}$$ and assuming that $$\sigma^2=V=t$$, I get $$P(x)=\frac{e^{lnx/{2bt}}2\sqrt{blnx}}{x\sqrt{2{\pi}t}}$$ Is this right? Shouldn't I get a Gaussian? Also, was I right to take the values of x to be [0,t] and not ##(-\infty,+\infty)## ? Thank you in advance :)
 
Last edited by a moderator:
Physics news on Phys.org
Assassinos said:
I get $$P(x)=\frac{e^{lnx/{2bt}}2\sqrt{blnx}}{x\sqrt{2{\pi}t}}$$ Is this right? Shouldn't I get a Gaussian?
There's no reason to expect a Gaussian. The random variable ##W_t## is Gaussian, but ##X_t##, being a non-linear transformation of ##W_t##, is not.

Rather than wrestling with all those integrals, a simpler and more intuitive way to calculate the probability density is to first calculate the cumulative probability function, then differentiate.

Start with:

$$Pr(X_t < x) = Pr \left(\exp\left(-bW_t{}^2\right) < x\right)$$
then manipulate that until you get a probability that ##W_t## lies in a certain region (which will consist of the real line excluding an interval centred on zero). Then express that probability in terms of ##\Phi##, the cumulative probability function of the standard normal distribution.

Then differentiate that wrt ##x## and you'll have the probability density function.

There may be an alternative approach using Ito's Lemma, but you may not have studied that lemma yet, and I think it may take longer than the above anyway.
 
Last edited:
  • Like
Likes member 587159
Question: A clock's minute hand has length 4 and its hour hand has length 3. What is the distance between the tips at the moment when it is increasing most rapidly?(Putnam Exam Question) Answer: Making assumption that both the hands moves at constant angular velocities, the answer is ## \sqrt{7} .## But don't you think this assumption is somewhat doubtful and wrong?

Similar threads

  • · Replies 11 ·
Replies
11
Views
3K
  • · Replies 14 ·
Replies
14
Views
2K
  • · Replies 19 ·
Replies
19
Views
4K
Replies
5
Views
3K
  • · Replies 6 ·
Replies
6
Views
3K
Replies
1
Views
1K
  • · Replies 3 ·
Replies
3
Views
2K
Replies
2
Views
2K
Replies
3
Views
2K
Replies
9
Views
2K